The Student Room Group

Lamina



I used the following formula:



I got the x-coordinate correct but the y-coordinate I am getting wrong:

I just integrated (4x-x3)2 (between the limits 0 and 2) then multiplied by (1/8). I multiplied by 1/8 because the denominator = 2 x 4 (I found the integral of f(x) between those limits, to be 4 in earlier calculations)

My calculator does numerical integration and gets the same answer as me 128/105 but the mark scheme has 64/21 as the answer
Original post by GPODT

I just integrated (4x-x3)2 (between the limits 0 and 2) then multiplied by (1/8).


If you subtract one from the other, that has the effect of moving the area to the x-axis.

You need to square them individually, and then subtract. So, you're, in effect working out the moment from the x-axis to the upper line, and subtracting the moment from the x-axis to the curve, leaving the moment of the desired area.
(edited 11 years ago)

Quick Reply

Latest